RMO 400 points?

x 3 ( y 3 + z 3 ) = 2012 ( x y z + 2 ) {x^{3}(y^{3}+z^{3})=2012(xyz+2)}

Find the sum of all values of x , y , z x, y, z , such that there exists some positive integer triples ( x , y , z ) (x,y,z) such that x y z x \le y \le z and satisfy the equation above.


The answer is 505.

This section requires Javascript.
You are seeing this because something didn't load right. We suggest you, (a) try refreshing the page, (b) enabling javascript if it is disabled on your browser and, finally, (c) loading the non-javascript version of this page . We're sorry about the hassle.

1 solution

Solution. First note that x divides 2012 · 2 = 2^3 · 503. If 503 | x then the right-hand side of the equation is divisible by 503^3, and it follows that 503^2 | xyz + 2. This is false as 503 | x. Hence x = 2^m with m ∈ {0,1,2,3}. If m ≥ 2 then 2^6 | 2012(xyz + 2). However the highest powers of 2 dividing 2012 and xyz+2 = (2^m)(yz)+2 are 2^2 and 2^1 respectively. So x = 1 or x = 2, yielding the two equations

y^3 +z^3 =2012(yz+2), and y^3 +z^3 =503(yz+1).

In both cases the prime 503 = 3 · 167 + 2 divides y^3 + z^3. We claim that 503 | y + z. This is clear if 503 | y, so let 503 ∤ y and 503 ∤ z. Then y^502 ≡ z^502 (mod 503) by Fermat’s little theorem. On the other hand y^3 ≡ −z^3 (mod 503) implies y^3·167 ≡ −z^3·167 (mod 503), i. e. y^501 ≡ −z6501 (mod 503). It follows that y ≡ −z (mod 503) as claimed. Thereforey+z=503k with k≥1. In view of y^3+z^3 =(y+z)􏰁((y−z)^2+yz) 􏰂the two equations take the form k(y−z)^2 +(k−4)yz=8, (1) k(y−z)^2 +(k−1)yz=1. (2) In (1) we have (k−4)yz≤8, which implies k≤4. Indeed if k>4 then 1≤(k−4)yz≤8, so that y≤8 and z≤8. This is impossible as y+z=503k≥503. Note next that y^3+z^3 is even in the first equation. Hence y + z = 503k is even too, meaning that k is even. Thus k=2 or k=4. Clearly(1) has no integer solutions for k=4. If k=2 then(1) takes the form (y + z)^2 − 5yz = 4. Since y + z = 503k = 503 · 2, this leads to 5yz = 5032 · 22 − 4. However 5032 · 22 − 4 is not a multiple of 5. Therefore (1) has no integer solutions. Equation(2) implies 0≤(k−1)yz≤1, so that k=1 or k=2. Also 0≤k(y−z)2 ≤1, hence k=2 only if y=z. However then y=z=1,which is false in view of y+z≥503. Therefore k=1 and (2) takes the form (y−z)^2 =1, yielding z−y=|y−z|=1. Combined with k = 1 and y + z = 503k, this leads to y = 251, z = 252. In summary the triple (2, 251, 252) is the only solution.

0 pending reports

×

Problem Loading...

Note Loading...

Set Loading...